Schwache Wechselwirkung verletzt die konjugierte Ladung

Wie können wir zeigen, dass die schwache Wechselwirkung die Ladungskonjugationssymmetrie verletzt?

Fragen Sie nach einem Beweis im mathematischen Formalismus oder fragen Sie nach experimentellen Beweisen? Und meinst du nur die Ladungskonjugation? C , oder die Materie-Antimaterie-Konjugation C P ? Die erste ist irgendwie lahm – wir wissen, dass schwache Wechselwirkungen stark verletzen P , aber ungefähr erhalten T , also müssen sie verletzen C konservieren C P T . Die Tatsache, dass das Universum eine Materie-Antimaterie-Asymmetrie aufweist, impliziert dies C P Verletzung, die ein aktives Forschungsgebiet ist.

Antworten (2)

Schwache Wechselwirkungen umfassen nur die linken Neutrinos (und die rechten Antineutrinos). Das bedeutet, dass alle Neutrino-Wechselwirkungsterme in der Lagrangefunktion auch nur aus linken Teilchen (und rechten Antiteilchen) bestehen, weil Ψ ¯ γ μ Φ R , L = Ψ ¯ R , L γ μ Φ R , L . Es bedeutet, dass die berechneten aktuellen Bedingungen L C C = G l ¯ L γ μ ( v l ) L W μ bricht die C-Invarianz: Neutrino wechselwirkt nur mit Lepton, während das Antineutrino nur mit Antilepton wechselwirkt.

Es kann leicht aus der Definition der Ladungskonjugationsoperation in den Raum von Darstellungen vom Dirac-Typ verstanden werden. Für die beliebige halbzahlige Spin-Funktion

Ψ μ 1 . . . μ N = ( Ψ A   μ 1 . . . μ N Φ A ˙   μ 1 . . . μ N ) ,
(hier entspricht die Zahl des Vektorindex dem ganzzahligen Teil des Spinwerts; Dirac 1 2 Spinor hat null Vektorindizes)
C ^ Ψ μ 1 . . . μ N = ( Φ A   μ 1 . . . μ N Ψ A ˙   μ 1 . . . μ N ) .
Wie gezeigt werden kann, C ^ = γ 2 K (Ich habe die Phase vernachlässigt), also
1 ± γ 5 2 C ^ Ψ = ( 1 ± γ 5 2 ) γ 2 K Ψ = γ 2 K ( 1 γ 5 2 ) Ψ =
C ^ ( 1 γ 5 2 ) Ψ .
Hier habe ich die Dirac-Darstellung der Gamma-Matrizen (oder Wiederholungen, die damit durch die Orthogonalitätsmatrix verbunden sind) verwendet, in denen γ 5 = γ 5 . Auch habe ich Gleichheit verwendet [ γ 5 , γ μ ] + = 0 die in jeder Darstellung enthalten ist. Die endgültige Gleichheit bedeutet, dass, wenn wir durch den Chiralitätsprojektoroperator auf den C-invertierten Spinor einwirken, der linke Projektor als rechter Projektor fungiert, während der rechte Projektor als linker fungiert. Das bedeutet es C ^ ändert den Eigenzustand des Chiralitätsprojektors in den "entgegengesetzten".

Dies ist das Partikelergebnis der in dieser Antwort gegebenen Aussage.

Über den C-Konjugationsoperator.

Der C-Operator vertauscht im Allgemeinen die Funktion von der linken Darstellung des Spinors vom Dirac-Typ zur rechten Darstellung. Wir müssen diese spezifische Struktur konstruieren, weil die irreduzible Darstellung der Poincare-Gruppe für halbzahligen Spin ihre eigenen Besonderheiten hat (ich möchte diese Aussage nicht verfeinern, da sie viel Platz einnehmen wird; in wenigen Worten, für halbzahlige Spin-Darstellung können wir nicht nur 4-Vektor-Indizes verwenden). Dieser Operator kombiniert die komplexe Konjugation (die für Integer-Spin-Darstellungen gleich der Ladungskonjugation ist) und die "Konjugation" der linken und rechten Darstellung.

Wie gezeigt werden kann, ändert dieser Operator auch den zusammenfassenden Ladungswert des dirac-Typ-Feldes. Dh, wenn wir das Konservierte konstruieren U ( 1 ) Strom und dann mit dem Ladungskonjugationsoperator darauf einwirken, ändern wir das Vorzeichen dieses Operators (also ändern wir insbesondere die elektrische Ladung).

Nehmen wir das einfachste Beispiel – den Dirac-Spinor Ψ = ( ψ A κ A ˙ ) T . Als Irrep der Lorentz-Gruppe kann es konstruiert werden als ( 1 2 , 0 ) ( 0 , 1 2 ) (Der erste bezieht sich auf die ψ A während sich das zweite auf das Komplexkonjugierte bezieht κ A ˙ ). Der Ladungskonjugationsoperator (hier habe ich die übliche Phase wiederhergestellt ich ) gibt

C ^ Ψ = ich γ 2 Ψ = ( κ A ψ A ˙ )
Der Dirac-Spinorstrom ist gleich J μ = Ψ ¯ γ μ Ψ , also wenn wir handeln J 0 durch den C-Operator, werden wir geben
J C 0 = ( ( C ^ Ψ ) C ^ Ψ ) = Ψ T γ 2 γ 2 Ψ = Ψ Ψ .
Hier habe ich die Tatsache ausgenutzt, dass Spinoren Grasmanianer sind, also Ψ T Ψ = Ψ Ψ .

Vielen Dank, ich habe die letzte Zeile nicht verstanden, könnten Sie sie erweitern.
@ user55944 : Entschuldigung, ich habe den Fehler bei Zwischenberechnungen gemacht. Ich werde es in ein paar Minuten korrigieren und eine Erklärung hinzufügen.
Was ist K? und da ist ich im mittleren Ausdruck möchte ich den C-Oper verstehen. mehr, z. C ( ich ) = ich ? Und C A A = A A ? und C(e) = -e?
@ user55944 : entschuldige mich noch einmal. K ist der komplexe Konjugationsoperator, ich ist nur der Druckfehler. Ich werde meiner Antwort auch weitere Informationen über die spezifische Struktur des C-Operators für Dirac-ähnliche Darstellungen hinzufügen.

Nach der Konvention von Peskin & Schroeder – Transformationsregel von Dirac-Spinoren ψ ( X , T ) unter C Und P werden gegeben von,

C ψ ( T , X ) C = ich ( ψ ¯ γ 0 γ 2 ) T C ψ ¯ ( T , X ) C = ich ( γ 0 γ 2 ψ ) T
Lassen Sie uns studieren, wie v μ ψ ¯ γ μ ψ Transformation unter Ladung Konjugation C .

C v μ C = C ψ ¯ γ μ ψ C = C ψ ¯ C C γ μ C C ψ C angesichts   C = C 1 = ich ( γ 0 γ 2 ψ ) T γ μ ( ich ) ( ψ ¯ γ 0 γ 2 ) T = ψ ¯ γ μ ψ = v μ
Ebenso kann man das zeigen A μ ψ ¯ γ 5 γ μ ψ , verwandeln Sie sich unter C Betrieb als C A μ C = A μ . Betrachten wir die Lagrange-Funktion der schwachen Wechselwirkung.
L schwach G F 2 ( v μ A μ ) ( v μ A μ ) = G F 2 ( v 2 2 v μ A μ + A 2 )
Es genügt zu studieren, wie v 2 2 v μ A μ + A 2 unter transformieren C um die Invarianz der Lagrangefunktion der schwachen Wechselwirkung unter Ladungskonjugation zu überprüfen.
v 2 2 v μ A μ + A 2 C v 2 + 2 v μ A μ + A 2 L schwach
Daher ist der Lagrangian der schwachen Wechselwirkung unter Ladungskonjugation nicht invariant.